Download Problem 1. A cylinder in a magnetic field (Jackson)

Survey
yes no Was this document useful for you?
   Thank you for your participation!

* Your assessment is very important for improving the workof artificial intelligence, which forms the content of this project

Document related concepts

Maxwell's equations wikipedia , lookup

Electric dipole moment wikipedia , lookup

Magnetic stripe card wikipedia , lookup

Electromotive force wikipedia , lookup

Neutron magnetic moment wikipedia , lookup

Magnetic monopole wikipedia , lookup

Electromagnetism wikipedia , lookup

Magnetometer wikipedia , lookup

Mathematical descriptions of the electromagnetic field wikipedia , lookup

Giant magnetoresistance wikipedia , lookup

Friction-plate electromagnetic couplings wikipedia , lookup

Skin effect wikipedia , lookup

Superconducting magnet wikipedia , lookup

Earth's magnetic field wikipedia , lookup

Multiferroics wikipedia , lookup

Magnetotactic bacteria wikipedia , lookup

Electromagnetic field wikipedia , lookup

Magnet wikipedia , lookup

Magnetotellurics wikipedia , lookup

Magnetoreception wikipedia , lookup

Magnetism wikipedia , lookup

Lorentz force wikipedia , lookup

Ferrofluid wikipedia , lookup

Force between magnets wikipedia , lookup

Magnetochemistry wikipedia , lookup

Electromagnet wikipedia , lookup

Ferromagnetism wikipedia , lookup

History of geomagnetism wikipedia , lookup

Transcript
Problem 1.
A cylinder in a magnetic field (Jackson)
A very long hollow cylinder of inner radius a and outer radius b of permeability µ is placed
in an initially uniform magnetic field Bo at right angles to the field.
(a) For a constant field Bo in the x direction show that Az = Bo y is the vector potential.
This should give you an idea of a convenient set of coordinates to use.
Remark: See Wikipedia for a list of the vector Laplacian in all coordinates. Most often
the vector Laplacian is used if the current is azimuthal and solutions may be looked
for with A 6= 0 and Ar = A✓ = 0 (or A⇢ = Az = 0 in cylindrical coordinates). This
could be used for example in Problem 3. Similarly if the current runs up and down,
with Az 6= 0 and A⇢ = A = 0, so that B = (Bx (x, y, z), By (x, y, z), 0) is independent
of z, then the vector Laplacian in cylindrical coordinates r2 Az is a good way to go.
(b) Show that the magnetic field in the cylinder is constant ⇢ < a and determine its
magnitude.
(c) Sketch |B|/|Bo | at the center of the as function of µ for a2 /b2 = 0.9, 0.5, 0.1 for µ > 1.
1
Problem 2.
Helmholtz coils (Jackson)
Consider a compact circular coil of radius a carrying current I, which lies in the x
with its center at the origin.
y plane
(a) By elementary means compute the magnetic field along the z axis.
(b) Show by direct analysis of the Maxwell equations r · B = 0 and r ⇥ B = 0 that
slightly o↵ axis near z = 0 the magnetic field takes the form
Bz '
(Bzo )
1
2
+
2
@ 2 Bzo
@z 2
⌘
0
⇣
where 0 =
and 2 =
origin. For later use give 0 and
the loop.
2
1 2
⇢
2
z2
,
B⇢ '
2 z⇢ ,
(1)
are the field and its z derivatives evaluated at the
explictly in terms of the current and the radius of
Remark: The magnetic field near the origin satisfies r ⇥ B = 0, so r · B = 0. We
say it is harmonic function1 . Because the function is harmonic, the taylor series of B
on the z axis, is sufficient to determine the taylor series close to the z axis.
(c) Now consider a second identical coil (co-axial with the first), having the same magnitude and direction of the current, at a height b above the first coil, where a is the radii
of the rings. With the coordinate origin relocated at the point midway between the
two centers of the coils, determine the magnetic field on the z-axis near the origin as
an expansion in powers of z to z 4 . Use mathematica if you like. You should find that
the coefficient of z 2 vanishes when b = a
Remark For b = a the coils are known as Helmholtz coils. For this choice of b the z 2
terms in part (c) are absent. (Also if the o↵-axis fields are computed along the lines
of part (b), they also vanish.) The field near the origin is then constant to 0.1% for
z < 0.17 a.
1
This means that B can be written B =
r
where
2
r2
=0
Problem 3.
A magnetized sphere and a circular hoop
A uniformly magnetized sphere of radius a centered at origin has a permanent total magnetic
moment m = m ẑ pointed along the z-axis (see below). A circular hoop of wire of radius b
lies in the xz plane and is also centered at the origin. The hoop circles the sphere as shown
below, and carries a small current Io (which does not appreciably change the magnetic field).
The direction of the current Io is indicated in the figure.
z
Io
y
x
(a) Determine the bound surface current on the surface of the sphere.
(b) Write down (no long derivations please) the magnetic field B inside and outside the
magnetized sphere by analogy with the spinning charged sphere disucssed in class.
(c) Show that your solution satisfies the boundary conditions of magnetostatics on the
surface of the sphere.
(d) Compute the net-torque on the circular hoop. Indicate the direction and interpret.
3
Solution
(a) The surface current is given by the discontinuity in the magnetization
Kb
= n ⇥ (Mout
c
Min )
(8)
Then since Mout = 0 we have
Kb
=
c
M (n ⇥ ẑ)
(9)
where M = m/( 43 ⇡a3 ) is the magnetic dipole moment per volume. With n = r̂ and
ˆ + cos ✓r̂, we find
ẑ = sin ✓✓
Kb
3
=
m sin ✓ ˆ
c
4⇡a3
(10)
(b) A formal analogy with the charged spinning sphere gives the solution. In the spinning
sphere case the surface current was also / sin ✓ ˆ . In the spinning sphere case we
found that the magnetic field outside is one of a magnetic dipole, where all of magnetic
moment is placed at the origin
B=
Or more explicitly
B=
1
[3(m · r̂)r̂
4⇡r3
m]
1
1
ˆ.
2m cos ✓ r̂ +
m sin ✓ ✓
3
4⇡r
4⇡r3
(11)
(12)
Inside sphere, the magnetic field was constant
B = Bo ẑ
(13)
The constant Bo can be picked o↵ from the boundary conditions as we will do in the
next item.
(c) The boundary conditions read
n ⇥ (H2
n · (B2
H1 ) =0
B1 ) =0
(14)
(15)
Then from the second boundary condition at r = a
Br |out = Br |in .
(16)
With the magnetic field outside the sphere
Br |out =
5
1
2m cos ✓ ,
4⇡r3
(17)
and inside the sphere
r̂ · B|in = Bo r̂ · ẑ = Bo cos ✓ ,
comparison at r = a gives
(18)
1
2m .
(19)
4⇡a3
B1 ) = 0 is satisfied, provided Bo takes a specific
Bo =
Thus we have shown that n · (B2
value.
Note that inside the sphere with M = m/(4⇡a3 /3) and the inductance is
Hz,in = Ho = Bo
M=
m
.
4⇡a3
(20)
Now one can verify the parallel boundary condition n ⇥ (H2 H1 ) = 0 using eq. (20)
✓
◆
1
H✓,out H✓,in |r=a =
m sin ✓ + Ho sin ✓
=0
(21)
4⇡r3
r=a
(d) To compute the torque we first compute the lorentz force on a element of length
d` = bd✓.
Io
d`B?
c
Io
= bd✓ Br
c
Io
2m cos ✓
= bd✓
c
4⇡b3
dF =
(22)
(23)
(24)
The right hand rule indicates that the force is in the ŷ direction in the upper hemisphere, and in the positive ŷ direction in the lower hemisphere. This implies that the
net torque points along the x-axis. This can be intuited by noting that the magnetic
moment of the hoop tends to align with the magnetic field from the sphere
(e) The torque around the x-axis
⌧=
Z
Z
d⌧ =
⇡
Z
b cos ✓ dF
Io
2m cos ✓
bd✓
c
4⇡b3
0
Z
4m(Io /c)b2 ⇡
=
d✓ cos2 ✓
4⇡b3
0
4m(Io /c)b2 ⇡
=
4⇡b3
2
2m Io 2
=
⇡b
4⇡b3 c
=2
b cos ✓
6
(25)
(26)
(27)
(28)
(29)
Problem 4.
Energy of a wire and rectangle (Jackson)
(a) Consider an infinitely long straight wire carying a current I in the z direction. Use the
known mangetic field of this wire, and the integral form of B = r ⇥ A
Z
I
B · dS = d` · A
(2)
S
to show that the vector potential for an infinite current carrying wire in the Coulomb
gauge is
(I/c)
Az =
log ⇢ + const
(3)
2⇡
Check that the Coulomb gauge condition is satisfied.
(b) Now consider a flat right rectangular loop carrying a constant current I1 that is placed
near a long straight wire carrying a constant current I2 . The rectangular loop is
oriented so that its center is a perpendicular distance d from the wire; the sides of
length a are parrel to the wire and the sides of length b make an angle ↵ with the plane
containing the wire and the loops center (the dashed line below). In the schematic
diagram below, the current I2 in the long wire flows out of page. The orientation of I1
is also indicated, i.e. the current lower edge of the rectangle (of length a) also comes
out of the page.
↵
b/2
d
Show that the interaction energy
W12 =
I1
F1
c
(4)
(where F1 is the magnetic flux from I2 through the rectangular circuit carrying I1 ), is
 2
aI1 I2
4d + b2 + 4db cos ↵
W12 =
ln
(5)
4⇡c2
4d2 + b2 4db cos ↵
4
(c) Using energy considerations calculate the force between the loop and the wire for
constant currents.
(d) Check that for large distances d
a, b the force computed in the previous sub-question
agrees with the appropriate formula for a dipole in an external field.
(e) Show that when d
a, b the interaction energy reduces to W12 = m · B, where m is
the magnetic moment of the loop. Explain the sign.
5
Problem 5.
exam)
A half submerged metal sphere (UIC comprehensive
A very light neutral hollow metal spherical shell of mass m and radius a is slightly submerged
by a distance b ⌧ a below the surface of a dielectric liquid. The liquid has mass density ⇢
and electrical permitivity ✏. The liquid sits in air which has negligble density ⇢o ⌧ ⇢, and
the permitivity of air is approximately unity, ✏air ' 1. The pressure at the air liquid interface
is p0 . Recall that stress tensor of an ideal fluid at rest is T ij = p(z) ij where p(z) is the
pressure as a function of z.
Air
b
Liquid
(a) Use the formalism of stress tensor to show that p(z) increases as p = p0 + ⇢gh, where
h = z is the depth below the surface, z < 0. Here p0 is the pressure at the surface.
Hint: what is the net force per volume for a static fluid?
(b) Use the formalism of stress tensor to prove that the boyancy force (for any shape) equals
the di↵erence in weight of the displaced fluid volume V and the corresponding weight
of the air:
F = (⇢ ⇢o )g V ' ⇢g V.
(c) (Optional) Determine the boyancy force in this case.
Now a charge Q is added to the sphere, and the sphere becomes half submerged.
(d) Determine the potential, and the electrostatic fields E and D, in the top and lower
halves of the sphere. Verify that all the appropriate boundary conditions are satisfied.
(e) What is the surface charge density on the top and lower halves of the sphere?
(f) Use dimensional reasoning and all reasonable approximations to show that for a light
sphere to be half submerged we must have
p
Q = ⇢ga5 ⇥ function of ✏ .
(6)
(g) Determine the electrostatic attractive force as a function of Q, a, and ✏. What must Q
be for the sphere to be half submerged? Make all reasonable approximations. Express
your approximate result in terms of ⇢, g, a, ✏.
(h) (Optional) Estimate Q numerically for typical liquids.
6